Kann ΔxΔx\Delta x (σxσx\sigma_x) jedes freien Teilchenwellenpakets jederzeit abnehmen?

Stellen Sie sich ein beliebiges Wellenpaket vor, das ein freies Teilchen beschreibt (also kein Potential oder andere Kräfte, die darauf einwirken). Dann kann man das zeigen Δ p ändert sich zeitlich nicht. Meine Frage ist jedoch, was mit passiert Δ x wie wir in der Zeit voranschreiten? Muss es immer steigen? Oder gibt es ein Gegenbeispiel, bei dem die Positionsunsicherheit abnimmt, wenn auch nur für kurze Zeit?

Meine erste Vermutung ist Δ x muss immer zunehmen, weil p 0 , so dass Δ p 0 und daher Δ v = Δ p m 0 . Aber wenn es eine Ausbreitung der Geschwindigkeiten gibt, dann muss sich auch das Wellenpaket ausbreiten. Ist diese Logik richtig? Oder könnten wir ein Wellenpaket haben, bei dem sich das hintere Ende schneller vorwärts bewegt als das vordere, und für eine gewisse Zeit, bis das hintere Ende das vordere einholt, wäre es tatsächlich schmaler als am Anfang, dh abnehmend Δ x ? Wenn ja, wie würde man ein solches freies Teilchen (Wellenpaket) beschreiben?

Es scheint mir also, dass sich jedes Wellenpaket, das ein freies Teilchen beschreibt, irgendwann ausbreiten wird, aber die Frage ist, ob es einen Zeitraum in seiner Entwicklung geben kann, in dem es tatsächlich schmaler wird.

edit: Insbesondere wenn es nicht ständig steigen muss, kann dies gezeigt werden, ohne auf Zeitumkehr zu berufen?

Verwandte, aber nicht dieselbe Frage: physical.stackexchange.com/q/7231
@DavidZaslavsky Ja, das habe ich schon gesehen, und etwas Ähnliches wie meine Frage wurde dort auch in den Kommentaren diskutiert. Es wurde jedoch nie gesagt, ob Δ x muss immer steigen (oder im Idealfall gleich bleiben) , nicht nur irgendwann .

Antworten (4)

Wenn Ψ ( x , t ) löst die Schrödinger-Gleichung, tut dies auch Ψ ( x , t ) , also nein, es gibt überhaupt nichts, was zunehmen muss.

Können Sie das näher erläutern? Wollen Sie damit sagen, dass, wenn die Ungewissheit zunimmt, sagen wir, t ( t 1 , t 2 ) , dann gibt es einen Zeitraum, in dem die Positionsunsicherheit abnimmt, nämlich t ( t 2 , t 1 ) ?
Ich sage das, wenn Ψ ( x , t ) hat von t1 bis t2 zunehmende Unsicherheit, dann definieren Ψ ¯ ( x , t ) = Ψ ( x , t ) , und Ψ ¯ wird von t1 zu t2 eine abnehmende Unsicherheit aufweisen.
Ich finde es sehr offensichtlich, um ehrlich zu sein. Ich sage nur, dass Sie alles zeitlich umkehren können. Wenn etwas steigt, wenn die Zeit vorwärts geht, sinkt es, wenn die Zeit rückwärts geht, aber die Schrödinger-Gleichung kann den Unterschied zwischen "Zeit geht vorwärts" und "Zeit geht rückwärts" nicht erkennen, also kann alles, was für eine Lösung zunehmen kann, abnehmen für eine andere Lösung.
Tolles, einfaches Argument. Ich wünschte, ich hätte nicht so viel Zeit mit dem Rechnen verbracht.
Hmm, verstehe, aber es lässt sich zum Beispiel nachweisen, dass die Unsicherheit in der Position eines Gaußschen Wellenpakets immer größer wird. Wenn Sie also dieses Wellenpaket zeitlich umkehren und es so definieren, wie Sie es in Bezug auf gemacht haben Ψ ( x , t ) , ist dieses zeitumgekehrte Wellenpaket dann nicht immer noch Gaußsch, was zu einem Widerspruch führt?
Ich sehe keinen Widerspruch, aber ich verstehe auch nicht wirklich, warum Sie denken, dass es einen gibt. Wenn Sie sagen "wenn Sie dieses Wellenpaket zeitlich umkehren", ist das Wort "das" mehrdeutig; Ich weiß nicht genau, welche Wellenfunktion Sie zeitlich umkehren möchten. Vielleicht rührt Ihre Verwirrung daher, dass Sie denken, dass eine Gaußsche Funktion eine Gaußsche Funktion bleibt, wenn sie sich mit der Zeit weiterentwickelt. Er tut es nicht, denn wenn er es täte, würde sich die Ungewissheit im Widerspruch zu Ihrer eigenen Aussage ändern (Gaußsche Sättigung der Unschärferelation).
OK, verstehe, und danke für das zusätzliche Feedback. Zwei Dinge verstehe ich allerdings noch nicht ganz. Erstens, in Ihrer Antwort sagen Sie das Ψ ( x , t ) löst die Schrödinger-Gleichung, aber ich verstehe nicht, wie das offensichtlich ist. Wie zeigt man das? Und zweitens verstehe ich nicht, wie die Gaußsche aufhört, eine Gaußsche zu sein. Ich meine, Sie setzen im Grunde einfach einen bestimmten Wert für t für spätere Zeiten ein, und Sie haben immer noch einen Ausdruck derselben Form, aber jetzt anders x 0 oder p 0 oder was auch immer der Gipfel auszeichnet, nicht wahr?
Was die Evolution eines Gaußschen betrifft, nein, wie ich bereits gezeigt habe, ist das unmöglich. Gehen Sie einfach die Gleichung lösen.
@MarkEichenlaub Was diesen Wikipedia-Artikel betrifft, heißt es selbst, dass man ihn möglicherweise nur in bestimmten Kontexten findet. Warum sollten Sie ihn dann unbedingt auf die Schrödinger-Gleichung anwenden? Gibt es insbesondere eine Möglichkeit zu zeigen, was Sie gesagt haben, indem Sie die Schrödinger-Gleichung direkt verwenden, ohne sich auf eine allgemeine theoretische Vorhersage wie diese zu berufen? Wenn ja, würde ich mich über weitere Details dazu sehr freuen.
Natürlich ist die Schrödinger-Gleichung zeitumkehrbar, obwohl es ein Detail gab, das ich ursprünglich in meiner Antwort ausgelassen hatte. Siehe physical.stackexchange.com/questions/40996/…
Ich werde versuchen, mich um diese Zeitumkehr-Sache zu kümmern, aber gibt es in der Zwischenzeit eine Möglichkeit, das zu zeigen, was Sie ohne einen solchen Rückgriff gesagt haben? Diese Frage wurde nämlich in unserem ersten QM-Kurs gestellt, aber nie beantwortet, und ich gehe davon aus, dass es eine Antwort nur mit diesen Tools geben muss. Und ich muss sagen, dass der Ansatz, den @joshphysics gewählt hat, auch der ist, an den ich zuerst gedacht habe. Irgendwie würde das funktionieren?
Ich verstehe die Frage nicht. Willst du ein Beispiel? Nehmen Sie einfach eine beliebige Wellenfunktion, entwickeln Sie sie mit der Zeit ein wenig weiter und sehen Sie, ob ihre Unsicherheit abnimmt. Wenn nicht, wenden Sie die Transformation an, entwickeln Sie sie zeitlich etwas weiter, und Sie haben ein konkretes Beispiel.
Lösung gegeben Ψ ( t ) der Schrödinger-Gleichung, Stecker Φ ( t ) = Ψ ( t ) in die Gleichung, nimm ihr komplexes Konjugat, und du wirst sehen, dass es eine Lösung ist. Wenn Ψ Zunahmen in der Breite zwischen t 1 und t 2 , dann Φ wird in der Breite zwischen abnehmen t 2 und t 1 .
Ich denke, was ich wissen wollte, ist, ob Sie das zeigen können, ohne anzunehmen, dass Sie die von Ihnen vorgeschlagene Transformation durchführen können.
Oder, wenn Sie ein einfach zu visualisierendes Beispiel wünschen, nehmen Sie als Beispiel zwei Gaußsche mit einem sehr weiten Abstand, die sich aber aufeinander zu bewegen. Dann ist die Unsicherheit in x im Grunde nur die Trennung, und die Streuung der Gauß-Funktionen wird ignoriert. Entwickeln Sie das mit der Zeit etwas weiter. Die Gaußschen kommen näher zusammen, wodurch die Unsicherheit in x verringert wird, obwohl sie sich einzeln etwas ausbreiten.
@EmilioPisanty, also ist es eigentlich zwischen t 2 und t 1 . Ich habe das nämlich im ersten Kommentar oben gefragt, aber Mark hat geantwortet, dass es zwischendrin sein würde t 1 und t 2 . Das hat mich etwas verwirrt, also können wir uns da zufällig einigen?
Ich denke, das ist ein Ausrutscher von Mark (ohne Schuldzuweisungen, sollte ich meinen). In der Zwischenzeit setzen Sie sich hin und rechnen Sie!
@MarkEichenlaub Ich habe vorher an dieses Gaußsche Beispiel gedacht, aber würden diese beiden Gaußschen ein freies Teilchenwellenpaket beschreiben? Da war ich mir nicht sicher, und ich war mir auch nicht sicher, ob wir einfach mit der Hand winken können, um zu sagen, dass die Unsicherheit abnimmt, wenn sie näher zusammenkommen. Denn wer sagt, dass die Streuung, wenn die Zentren ausgerichtet sind, nicht noch größer ist als die Trennung zu Beginn?
Emilio hat Recht. Die Idee ist nur Zeitumkehr. Bitte entschuldigen Sie, aber die Einzelheiten interessieren mich nicht mehr; Dieser Kommentarthread ist schon zu lang.
Ja, die beiden Gaußschen beschreiben ein freies Teilchen. Wenn ein Gaußsches tut, können zwei Gaußsche durch Superposition. Es ist einfach, die Breite der Gaußschen und die Trennungen so anzupassen, dass die Aussage korrekt ist.
Danke, ich werde das später noch einmal durchgehen und die von @EmilioPisanty vorgeschlagene Berechnung durchführen, und ich werde einfach hierher zurückkehren, wenn ich noch Fragen habe oder etwas finde, das nicht mit dem übereinstimmt, was gesagt wurde.
  1. Wir formulieren die Titelfrage von OP (v1) wie folgt um:

    Zeigen Sie das für alle möglichen Wellenpakete ψ ( p , t ) eines freien Teilchens die Ortsvarianz

    (1) v a r ( x ^ )   =   x ^ 2 x ^ 2
    nimmt in mindestens einem Intervall ab [ t 1 , t 2 ] von Zeit.

    Wie Mark Eichenlaub in seiner Antwort und seinen Kommentaren richtig feststellt, eine Lösung mit Abnehmen v a r ( x ^ ) in [ t 1 , t 2 ] kann durch Zeitumkehrsymmetrie auf eine Lösung mit steigendem Wert abgebildet werden v a r ( x ^ ) in [ t 2 , t 1 ] . Hier wird verwendet, dass der Hamiltonoperator H ^ = p ^ 2 2 m für ein freies Teilchen und den Positionsoperator x ^ beide kommutieren mit dem Zeitumkehroperator T ^ . Siehe zB meine Phys.SE-Antwort hier für weitere Details.

    Die Zeitumkehrsymmetrie schließt jedoch logischerweise nicht allein die Möglichkeit aus, dass eine Lösung monoton steigend ist v a r ( x ^ ) für immer. (Es impliziert nur, dass es in diesem Fall auch eine monoton fallende Lösung geben wird v a r ( x ^ ) für immer.)

  2. Explizite Berechnung. Betrachten Sie ein beliebiges Wellenpaket

    (2) ψ ( p , t )   =   EIN ( p ) e ich θ ( p , t )
    die die allgemeine Lösung der zeitabhängigen Schrödinger-Gleichung in der Impulsdarstellung darstellt. Hier der Winkel
    (3) θ ( p , t )   =   θ 0 ( p ) + p 2 2 m t , EIN ( p ) , θ 0 ( p )     R ,
    ist affin in t . Wir nehmen an, dass das Wellenpaket normiert ist
    (4) 1   =   | | ψ ( t ) | | 2   =   ψ ( t ) | ψ ( t )   =   R d p   EIN 2 .
    Der Ortsoperator in der Impulsdarstellung lautet
    (5) x ^   =   ich p .
    Daher der Positionsdurchschnitt
    (6) x ^   =   ψ ( t ) | x ^ | ψ ( t )   =   R d p   EIN 2 θ '
    ist affin in t , und der Durchschnitt der quadrierten Position
    (7) x ^ 2   =   ψ ( t ) | x ^ 2 | ψ ( t )   =   2 R d p   ( EIN ' 2 + EIN 2 θ ' 2 )
    ist quadratisch ein t . Hier bezeichnen Primzahlen Differenzierung bzgl. p .

    Somit wissen wir sofort, dass die Positionsvarianz

    (8) v a r ( x ^ )   =   a t 2 + b t + c
    ist quadratisch ein t auch wo a , b , und c sind konstante Koeffizienten. Dies ist aus der Cauchy-Schwarz-Ungleichung leicht ersichtlich
    (9) ( R d p   EIN 2 p m ) 2     R d p   ( EIN p m ) 2 R d p   EIN 2
    dass der Koeffizient zweiter Ordnung
    (10) a   =   R d p   ( EIN p m ) 2 ( R d p   EIN 2 p m ) 2     0 ,
    ist nichtnegativ, vgl. die Normalisierungsbedingung (4). Tatsächlich darf man das zeigen a > 0 ist streng positiv, und daher die v a r ( x ^ ) ist eine abnehmende Parabel für t ausreichend negativ, wie wir zeigen wollten.

    Skizzierter indirekter Beweis von a 0 : Der Koeffizient zweiter Ordnung a wird Null     die Cauchy-Schwarz-Ungleichung (9) wird zu einer Gleichheit     EIN ( p ) p ist proportional zu EIN ( p )     EIN ( p ) ist proportional zu einer Delta-Funktion δ ( p p 0 ) . Dies entspricht aber keinem normierbaren Wellenpaket.

  3. Beispiel. Zwei sich einander nähernde Wellenzüge sind ein einfaches intuitives Beispiel für die Positionsvarianz v a r ( x ^ ) nimmt in einem bestimmten Zeitintervall ab [ t 1 , t 2 ] . Aber das ist in gewisser Weise ein faules Beispiel, das irgendwie verrät, wie universell und allgegenwärtig dieses Verhalten für die Quantenmechanik ist.

    Wie wir beispielsweise aus der allgemeinen Analyse in Abschnitt 2 wissen, zeigt bereits das einfachstmögliche Wellenpaket, also ein einzelnes Gaußsches Wellenpaket, dieses Verhalten. Das ist jedoch viel weniger intuitiv und daher umso faszinierender / verblüffender zu versuchen, es zu verstehen. Lassen Sie uns der Einfachheit halber festlegen = 1 = m .

    Ein einzelnes Gaußsches Wellenpaket bei t = 0 ist von der Form

    (11) ψ ( p , t = 0 )   =   N exp ( ich p c p 2 2 τ ) ,
    wo τ > 0 und c = a + ich b C sind Konstanten. Hier N > 0 ist eine Normalisierungskonstante. Die Normierungsbedingung (4) impliziert dies
    (12) N   =   ( τ π ) 1 4 exp ( b 2 2 τ ) .
    Also für beliebige Zeit t , lautet das Gaußsche Wellenpaket
    (13) ψ ( p , t )   =   ψ ( p , 0 ) exp ( ich p 2 2 t )   =   N exp ( ich p c p 2 2 ( τ + ich t ) ) .
    In der Positionsdarstellung wird das Gaußsche Wellenpaket
    (14) ψ ( x , t )   =   R d p 2 π exp ( ich p x ) ψ ( p , t )   =   N τ + ich t exp ( ( x c ) 2 2 ( τ + ich t ) ) .
    Die Positionswahrscheinlichkeitsverteilungen werden
    (fünfzehn) | ψ ( x , t ) | 2   =   N 2 | τ + ich t | exp ( R e ( x c ) 2 τ + ich t )   =   N 2 τ 2 + t 2 exp ( [ ( x a ) 2 b 2 ] τ 2 ( x a ) b t τ 2 + t 2 )   =   τ π ( τ 2 + t 2 ) exp ( ( x a b t τ ) 2 τ τ 2 + t 2 ) .
    Daher der Positionsdurchschnitt
    (16) x ^   =   R d x   x | ψ ( x , t ) | 2   =   a + b t τ
    ist eine affine Funktion von t , während die Positionsabweichung
    (17) v a r ( x ^ )   =   R d x   ( x x ^ ) 2 | ψ ( x , t ) | 2   =   τ 2 + t 2 2 τ
    ist eine quadratische Funktion von t .

    Das zeitsymmetrische Profil (17) der Positionsvarianz v a r ( x ^ ) eines einzelnen Gaußschen Wellenpakets ist wahrscheinlich für jeden etwas überraschend, der seine Intuition aus der klassischen Physik entlehnt.

Ich kenne die Antwort darauf noch nicht, aber hier ist eine Berechnung, die andere bei der Bestimmung der Antwort nützlich finden könnten. Lassen Sie uns die zeitliche Ableitung von berechnen σ X . Notieren Sie sich das zunächst

d d t σ X 2 = 2 σ X σ ˙ X , σ X = X 2 X 2
Also
σ ˙ X = 1 2 σ X d d t ( X 2 X 2 ) = 1 2 σ X ( d d t X 2 2 X d d t X )
Verwenden Sie nun die allgemeine Beziehung
d d t Ö = ich [ H , Ö ]
für einen Operator ohne explizite Zeitabhängigkeit. Es folgt dem
d d t X = ich [ P 2 / 2 m , X ] = ich 2 m 2 ich P = P m
also das auch
d d t X 2 = ich [ P 2 2 m , X 2 ] = ich 2 m ( 2 ich ) { P , X } = { P , X } m
wo { P , X } = P X + X P , und deshalb
σ ˙ X = 1 2 m σ X ( { P , X } 2 P X )
Wir wollen wissen, ob es einen Zustand gibt, für den irgendwann σ ˙ X < 0 . Seit σ X > 0 , der gerade hergeleitete Ausdruck führt uns zu einer äquivalenten Frage; Gibt es einen Zustand, für den die folgende Ungleichung irgendwann gilt?
{ P , X } < 2 P X ?
Lassen Sie mich wissen, wenn Sie irgendwelche Fehler in der Mathematik finden, da ich es schnell gemacht habe.

Sieht gut aus für mich. Wenden Sie den Ansatz von Mark Eichenlaub darauf an. Angenommen, es existiert ein solcher Zustand σ ˙ x > 0 . Dann nach deiner Rechnung 2 P X > { P , X } . Jetzt unter Zeitumkehr P ändert das Vorzeichen aber nicht X , also unter T: 2 P X > { P , X } Multiplizieren Sie nun mit einem Minus durch und Sie erhalten das gewünschte Ergebnis.
@MichaelBrown Cool danke. Irgendwie wäre ich mit einer Berechnung für einen bestimmten Zustand immer noch zufriedener, in der die Schranken explizit verifiziert würden. Es wäre schön zu sehen, wie es funktioniert.
Sie könnten einen Beispielzustand von zwei Gaußschen nehmen, die aufeinander zu kommen, mit x = 0 dazwischen. Dann ist <p> = <x> = 0, aber der quadratische Operator wird nicht durch die Spiegelbildsymmetrie getötet.
Wenn mein Gehirn nicht kaputt ist (heute nicht genug Kaffee), dann | 0 > | 2 > (in harmonischen Oszillator-Eigenfunktionen) sollte ein weiteres Beispiel sein. Nicht zu sicher, dass es funktioniert, aber wenn nicht, sollte es nicht schwer sein, ein Beispiel zu verwenden. :)
Derp derp funktioniert nicht. Zeigt, dass ich einen Kaffee machen und mich hinlegen muss.
Ich habe diese Berechnung jetzt auch durchgeführt, und es scheint genau so zu funktionieren, wie Sie es gezeigt haben. Und, @MarkEichenlaub, das ist ein schönes Beispiel, um zu zeigen, dass der RHS-Begriff verschwindet, obwohl ich mich frage, ob es offensichtlich ist, dass der LHS-Begriff nicht getötet wird oder in diesem speziellen Fall negativ ist. Das habe ich woanders gesehen x p = x p + p x 2 , und intuitiv, wenn Sie zwei Pakete haben, wie vorgeschlagen, dann wäre die klassische Multiplikation xp für beide Gaußschen entweder vor oder nach dem Erreichen des Ursprungs positiv, wodurch die Ungleichung erfüllt wird. Wäre das vernünftige Logik?

Es gibt noch eine andere Lösung (vielleicht elementarer) 1 , mit einigen Komponenten der Antworten von Qmechanic und JoshPhysics (derzeit besuche ich meinen ersten QM-Kurs und verstehe die Lösung von Qmechanic nicht ganz, und diese Antwort ergänzt die Antwort von JoshPhysics) verwendet die Lösung die Heisenberg-Gleichung:

Die Zeitentwicklung eines Operators EIN ^ im Heisenberg-Bild der Quantenmechanik ist gegeben durch:

d EIN ^ d t = 1 ich [ EIN ^ , H ^ ]

Für ein freies Teilchen mit H ^ = p ^ 2 2 m , die Zeitentwicklung der Operatoren x ^ und p ^ sind :

d p ^ ( t ) d t = 0 d x ^ ( t ) d t = p ^ m

Dies sind die Bewegungsgleichungen im Heisenberg-Bild eines freien Teilchens, jetzt ergeben die Lösungen die gleichen wie für ein klassisches freies Teilchen:

p ^ ( t ) = p ^ ( 0 ) x ^ ( t ) = x ^ ( 0 ) + p ^ ( 0 ) m t

Mit diesen Ausdrücken ist leicht zu sehen (oder verwenden Sie den Satz von Ehrenfest ),

x ^ ( t ) 2 = x ^ ( 0 ) 2 + 2 t m x ^ ( 0 ) p ^ ( 0 ) + t 2 m 2 p ^ ( 0 ) 2
x ^ ( t ) 2 = x ^ ( 0 ) 2 + t m { x ^ ( 0 ) , p ^ ( 0 ) } + x ^ ( 0 ) + t 2 m 2 p ^ ( 0 ) 2
p ^ ( t ) 2 = p ^ ( 0 ) 2
p ^ ( t ) 2 = p ^ ( 0 ) 2

Wo wir den Antikonmutator definiert haben als { x ^ ( 0 ) , p ^ ( 0 ) } = x ^ ( 0 ) p ^ ( 0 ) + p ^ ( 0 ) x ^ ( 0 ) , damit können wir die Standardabweichung der Impuls- und Ortsoperatoren des Teilchens berechnen, jetzt für den Ortsoperator:

( Δ x ^ ( t ) ) 2 = x ^ ( t ) 2 x ^ ( t ) 2 = ( Δ x ^ ( 0 ) ) 2 + t m ( { x ^ ( 0 ) , p ^ ( 0 ) } 2 x ^ ( 0 ) p ^ ( 0 ) ) + t 2 m 2 ( Δ p ^ ( 0 ) ) 2

Wir können sehen, dass es ein Ausdruck der Form ist, a t 2 + b t + c mit

a = ( Δ p ^ ( 0 ) ) 2 b = ( { x ^ ( 0 ) , p ^ ( 0 ) } 2 x ^ ( 0 ) p ^ ( 0 ) ) c = ( Δ x ^ ( 0 ) ) 2

Wir können das sehen a , c 0 , es gibt jedoch keine Einschränkungen für b , es kann im Prinzip jeden Wert annehmen, wo man sich einen Anfangszustand vorstellen kann b ist negativ und sehr groß für klein t , beginnt die Standardabweichung abzunehmen, aber schließlich für eine groß genug t es beginnt zu steigen, wie man in Abschnitt 5 dieses Artikels Wellenpaketausbreitung sehen kann: Temperatur- und Quetscheffekte mit Anwendungen zur Quantenmessung und Dekohärenz

Es ist interessant, dass wir mit irgendwie anderen Ansätzen zum gleichen Ergebnis wie Joshphysics kommen.

1 Ich poste diese Antwort erneut, um die bereits gegebenen zu ergänzen. Als ich persönlich auf dieses Problem stieß, war ich mit diesem Thread verwirrt. Ich habe ihn gelesen, aber ich habe die Zeitumkehrsymmetrie nicht ganz verstanden, und die anderen Lösungen waren verwirrend oder zu fortgeschritten für mein Niveau. Ich hoffe, dies gibt einem Schüler, der in Zukunft danach sucht, ein Licht. Eine gute Ergänzung für den Anfang findet sich hier Heisenberg Picture: U Colorado Advanced Quantum Mechanics